EM Case Files + SmartyPance

Pataasin ang iyong marka sa homework at exams ngayon gamit ang Quizwiz!

A 58 year-old male presents with chest pain. Vital signs include blood pressure of 210/175, pulse 80, RR 20. Which of the following would you expect to find on physical examination? A papilledema B carotid bruit C diastolic murmur D absent peripheral pulses

A papilledema

A 63 year-old female presents with a complaint of chest pressure for one hour, noticed upon awakening. She admits to associated nausea, vomiting, and shortness of breath. 12 lead EKG reveals ST segment elevation in leads II, III, and AVF. Which of the following is the most likely diagnosis? A Aortic dissection B Inferior wall myocardial infarction C Acute pericarditis D Pulmonary embolus

B Inferior wall myocardial infarction

An electrocardiogram (ECG) shows a sinus rhythm with varying T wave heights, axis changes every other beat and a wandering baseline. Which of the following is most likely the diagnosis? A Artifact B Digoxin toxicity C Pericardial effusion D

C Pericardial effusion

Which of the following is a cause of prerenal azotemia? A Infection B Renal toxins C Poor renal perfusion D Urinary tract obstruction

C Poor renal perfusion

What type of chest pain is most commonly associated with a dissecting aortic aneurysm? A Squeezing B Dull, aching C Ripping, tearing D Burning

C Ripping, tearing

Which of the following hypertensive emergency drugs has the potential for developing cyanide toxicity? A Sodium nitroprusside (Nipride) B Diazoxide (Hyperstat) C Labetalol (Normodyne) D Alpha-methyldopa (Aldomet)

A Sodium nitroprusside (Nipride)

An HIV-positive patient presents with worsening dementia, fever, headache, and right hemiparesis. MRI of the brain reveals six lesions throughout the brain that show ring enhancement and surrounding edema. Which of the following is the treatment of choice? A Sulfadiazine and pyrimethamine B Trimethoprim-sulfamethoxazole C Radiation therapy D Ventricular shunt placement

A Sulfadiazine and pyrimethamine

Contraindications to beta blockade following an acute myocardial infarction include which of the following? A Third degree A-V block B Sinus tachycardia C Hypertension D Rapid ventricular response to Atrial fibrillation/flutter

A Third degree A-V block

Which of the following is the most common cause of nontraumatic cardiac tamponade? A metastatic malignancy B uremia C acute idiopathic pericarditis D hemorrhage (anticoagulant use) E bacterial or tubercular pericarditis

A metastatic malignancy Less common causes of nontraumatic tamponade include acute or chronic idiopathic pericarditis, uremia, bacterial or tubercular pericarditis, hemorrhage (from anticoagulant use), systemic lupus erythematosus, radiation treatments, and myxedema.

A 48 year-old male presents to the ED with complaints of chest pressure, dyspnea on exertion, and diaphoresis that has been present for the last one hour. Electrocardiogram reveals normal sinus rhythm at 92/minute along with ST segment elevation in leads V3-V5. Initial cardiac enzymes are normal. What is the next most appropriate step in the management of this patient? A Coronary artery revascularization B Admission for medical management C Administer lidocaine D Administer nitrates

A

How should one measure when deciding on the appropriate size of An oral airway? A Corner of the mouth to the angle of the mandible B Corner of the mouth to the ear pinna C Middle of the mouth to the angle of the mandible D Middle of the mouth to the ear pinna

A

What organisms cause the majority of cases of urethritis? A Chlamydia trachomatis and Neisseria gonorrhoeae B Herpes simplex virus and human papillomavirus C Staphylococcus aureus and Streptococcus pyogenes D Trichomonas vaginalis and Ureaplasma urealyticum

A

Which of the following is the most common complication that occurs in the setting of acute pericarditis? A Pericardial effusion B Left ventricular failure C Superior vena cava syndrome D Subclavian steal syndrome

A

A 46 year-old male with no past medical history presents complaining of chest pain for four hours. The patient admits to feeling very poorly over the past two weeks with fever and upper respiratory symptoms. The patient denies shortness of breath or diaphoresis. On examination the patient appears fatigued. Vital signs reveal a BP of 130/80, HR 90 and regular, RR 14. The patient is afebrile. Labs reveal a Troponin I of 10.33 ug/L (0-0.4ug/L). Cardiac catheterization shows normal coronary arteries and an ejection fraction of 40% with global hypokinesis. Which of the following is the most likely diagnosis? A myocarditis B pericarditis C hypertrophic cardiomyopathy D coronary artery disease

A myocarditis Myocarditis often occurs secondary to acute viral illness and causes cardiac dysfunction. Patients will commonly have a history of a recent febrile illness. Chest pain may mimic that of a myocardial infarction and Troponin I levels maybe elevated in one-third of patients. Contractile dysfunction is seen on catheterization and/or echocardiogram.

58 year-old male presents with chest pain. Vital signs include blood pressure of 210/175, pulse 80, RR 20. Which of the following would you expect to find on physical examination? A papilledema B carotid bruit C diastolic murmur D absent peripheral pulses

A papilledema

A 72-year-old smoker with a positive history of severe degenerative arthritis, diabetes, and CVD presents to your office complaining of bilateral leg pain that occurs after walking 200 yards. He reports that rest improves his symptoms. Which of the following would be appropriate? A Ankle/ brachial indices B MRI of the lumbar spine C Ultrasonography of the lower extremities D EMG of the lower extremities E Arteriogram of the lower extremities

A Ankle/ brachial indices

A 60-year-old male is brought to the ED complaining of severe onset of chest pain and interscapular pain. The patient states that the pain feels as though "something is ripping and tearing". The patient appears shocky; the skin is cool and clammy. The patient has an impaired sensorium. Physical examination reveals a loud diastolic murmur and variation in blood pressure between the right and left arm. Based on this presentation what is the most likely diagnosis? A Aortic dissection B Acute myocardial infarction C Cardiac tamponade D Pulmonary embolism

A Aortic dissection

Which of the following steps can reduce the pain of needle puncture and local anesthetic injection? A Applying a topical anesthetic prior to injection B Avoiding insertion of the needle at large pores or hair follicles C Cooling the local anesthetic agent D Inserting the needle with bevel down position

A Applying a topical anesthetic prior to injection

A 58-year-old man presents to your office 4 weeks after being hospitalized for MI. He is complaining of chest pain, fever, and multiple joint pain. Laboratory tests do not show an increase in cardiac enzymes. The most likely diagnosis is A Dressler's syndrome B Costochondritis C Meigs' syndrome D Recurrent MI E Pneumonia

A Dressler's syndrome Explanation: Dressler's syndrome, or postmyocardial infarction syndrome, occurs several days to several weeks after MI. The condition is characterized by chest pain, fever, pericarditis with a pericardial friction rub, pericardial effusion, pleurisy, pleural effusions, and multiple joint pain. The cause is thought to be an autoimmune response to the damaged myocardial tissue and pericardium. The difference between Dressler's syndrome and recurrent MI is difficult to determine; however, in Dressler's syndrome, there is minimal or no increase in cardiac enzymes. Treatment includes the use of aspirin, NSAIDs, and, in some cases, corticosteroids.

A 40-year-old right-hand dominant man presents with curvilinear puncture wounds over his fourth and fifth metacarpal phalangeal joints on his right hand. Which of the following organisms are you concerned about and want to make sure antibiotics you prescribe will cover? A Eikenella B Pasteurella C Pseudomonas D Staph aureus

A Eikenella

A 35-year-old female presents with multiple ulcerative lesions on her labia and perineum. A Tzanck preparation of one of the lesions reveals multinucleated giant cells. Which of the following is the most likely diagnosis? A Herpes Simplex Virus (HSV) B Molluscum Contagiosum Virus (MCV) C Human Papillomavirus (HPV) D Syphilis

A Herpes Simplex Virus (HSV)

A 4 year-old boy is sent home from day care for a severe cough following one week of cold symptoms, including sneezing, conjunctivitis, and nocturnal cough. He presents with paroxysms of cough followed by a deep inspiration, and occasional post-tussive emesis. During severe paroxysms, he exhibits transient cyanosis. What is the most appropriate treatment for exposed contacts at his day care center? A Macrolide prophylaxis B Isolation C Observation and treatment only if symptomatic D Supportive care only

A Macrolide prophylaxis

A 64 year-old male, with a long history of COPD, presents with increasing fatigue over the last three months. The patient has stopped playing golf and also complains of decreased appetite, chronic cough and a bloated feeling. Physical examination reveals distant heart sounds, questionable gallop, lungs with decreased breath sounds at lung bases and the abdomen reveals RUQ tenderness with the liver two finger-breadths below the costal margin, the extremities show 2+/4+ pitting edema. Labs reveal the serum creatinine level 1.6 mg/dl, BUN 42 mg/dl, liver function test's mildly elevated and the CBC to be normal. Which of the following is the most likely diagnosis? A Right ventricular failure B Pericarditis C Exacerbation of COPD D Cirrhosis

A Right ventricular failure

A 22 year-old male presents to the clinic complaining of scrotal pain that radiates into the groin. Patient admits to being a weightlifter and was lifting 24 hours prior to this pain developing into the scrotum. The patient admits to being sexually active with only his male partner. Examination reveals a reddened scrotum and it is difficult to distinguish the epididymis from the testes on the right side. Elevation of the right testicle brings relief of the pain. This is known as a positive Prehn's sign. B Cullen's sign. C Rovsing's sign. D Murphy's sign.

A , Prehn's sign.

A 33-year-old woman is pregnant at 12 weeks' gestation and presents with right flank pain and gross hematuria. She is afebrile. Which of the following imaging tests is most appropriate for this patient? A Ultrasonography B KUB C IVP D Retrograde pyelography E Helical CT without contrast

A. Because the patient is pregnant during the first trimester, the initial imaging test should be sonography to avoid the radiation-related teratogenic/mutagenic effects on the fetus

A 52-year-old healthy jogger is brought to the ED following a syncopal episode. A diagnosis of ventricular tachycardia is made, and the patient is cardioverted. She states that she has had prior episodes of VT lasting less than 30 seconds each. What is the most appropriate treatment at this time? A No further therapy at this time B Amiodarone C β-blocker D Procainamide

A. Nonsustained VT is by definition a self-terminating event, and therefore usually no specific treatment is indicated at this time in the ED. Rather, treatment is directed at any existing heart condition. Thus, it is important for the patient to have a thorough evaluation by a cardiologist to assess the risk of recurrence and danger.

A 37-year-old woman presents with chest pain after smoking crack cocaine 2 hours ago. What are you most likely to see on the ECG? A Sinus tachycardia B SVT C VT D Atrial fibrillation

A.Although cocaine intoxication can cause multiple tachycardias, the most common regular rate tachycardia is sinus tachycardia. Thus, although it is most likely to be a sinus tachycardia, an ECG is important to obtain to ensure that there is not a more concerning rhythm.

A middle-age man had a witnessed collapse in a busy coffee shop. He was immediately assessed by bystanders and found to be pulseless and apneic. Cardiopulmonary resuscitation was initiated immediately by bystanders and continued by EMS personnel during his transport to the hospital emergency department. According to the report from the prehospital providers, the patient has been in and out of ventricular fibrillation. He received cardioversion and cardiopulmonary resuscitation (CPR) during his transport to your facility. The patient remained in pulseless ventricular fibrillation at the time of his arrival to the resuscitation room of the hospital. After endotracheal intubation, several rounds of medication administration and cardioversions, the patient had return of a faint pulse, blood pressure of 96/60 mm Hg, and was noted to be in sinus tachycardia on the monitor with a heart rate of 115 beats per minute. What additional treatments should be provided at this time?

Additional treatments: Coronary angiography with percutaneous interventions and cardiac electrophysiology evaluations have been shown to improve survival, as cardiac ischemia is the cause for the ventricular fibrillation in the majority of patients with out-of-hospital cardiac sudden death. Hypothermia therapy has been demonstrated to improve the neurologic outcomes in post-cardiac arrest survivors.

A 55-year-old man presents with chest pain described as sudden in onset and radiating to his back and then to his abdomen. His examination is notable for hypertension and abnormal pulses. You have high suspicion for aortic dissection and want to initiate therapy immediately while you confirm the diagnosis. Which medication should be initiated first? A Diltiazem B Esmolol C Nicardipine D Nitroglycerin

B

A 72-year-old female is being discharged from the hospital following an acute anterolateral wall myocardial infarction. While in the hospital the patient has not had any dysrhythmias or hemodynamic compromise. Which of the following medications should be a part of her discharge medications? A Warfarin (Coumadin) B Captopril (Capoten) C Digoxin (Lanoxin) D Furosemide (Lasix)

B

A 58-year-old man is brought into the ED by paramedics because of worsening dyspnea. He has CHF due to cardiovascular disease. On examination, his blood pressure is 150/100 mm Hg and heart rate is 104 beats per minute. He has jugular venous distention and rales in both lung fields. Which of the following is the most effective and most rapid method of reducing preload and afterload in this patient? A Diuretics B Nitroglycerin C Dobutamine D Morpine

B Nitroglycerin

A 26-year-old patient is brought to the emergency department after a head on collision. The patient complains of chest pain, dyspnea and cough. Examination reveals the patient to be tachypneic and tachycardic with a narrow pulse pressure. Jugular venous distension is noted. Electrocardiogram reveals nonspecific T-wave changes and electrical alternans. Which of the following is the most appropriate management plan for this patient? A Left tube thoracostomy B Pericardiocentesis C Fluid resuscitation D Immediate intubation

B Pericardiocentesis

Which of the following is a systemic manifestation of infective endocarditis? A Hemarthrosis B Petechiae C Cafe au lait spots D Bronzing of the skin

B Petechiae

A 60 year-old male with hypertension is brought to the emergency department 30 minutes after the sudden onset of severe chest pain that radiates to his back and arms. His blood pressure is 180/80 mmHg in his left arm; no blood pressure reading can be obtained from the right arm. ECG shows sinus tachycardia with left ventricular hypertrophy. A high pitched decrescendo diastolic murmur is heard along the left mid-sternal border. Which of the following is the most likely diagnosis? A Acute myocardial infarction B Aortic dissection C Pulmonary embolism D Right subclavian arterial embolus

B Aortic dissection

A 20-year-old man presents to the emergency department after being stung by a bee. His skin is red and covered with welts. He has obvious swelling of his lips and tongue but no wheezes, and he is able to open his mouth without difficulty. After treatment with appropriate medications, he complains of throat swelling, and his voice is hoarse. He has stridorous inspirations but a normal respiratory rate and effort and oxygen saturation. What is the most appropriate management of this patient's airway? A Continued observation as long as oxygen saturation remains normal. B Prepare for RSI. C Begin high-dose nebulized albuterol and continue to observe. D Cricothyroidotomy.

B Prepare for RSI. This patient displays signs of impending airway obstruction. His worsening airway edema, despite appropriate medical therapy, dictates that intubation should be performed quickly before complete airway occlusion. There is no wheezing to suggest bronchoconstriction that could be treated with a bronchodilator such as albuterol. Stridor is a worrisome sign of upper airway obstruction. Normal respiratory rate and oxygen saturation should not delay intubation, as falling oxygen saturation is a late sign of respiratory failure. Cricothyroidotomy is only indicated after all other measures have failed, or if there is such upper airway edema and distortion that intubation would be impossible. For instance, findings of tachypnea, use of accessory muscles, and drooling may indicate the need for cricothyroidotomy.

Which of the following is NOT an accurate description of hypothermia therapy for post-cardiac arrest patients? A Rapid cooling to reduce core temperature to 32-34°C for 24-48 hours B Therapeutic hypothermia has a proven effect in reducing the rate of recurrent ventricular fibrillation C Therapeutic hypothermia is associated with increased infections and bleeding complications D Therapeutic hypothermia is associated with some improved recovery of myocardial functions E The primary benefit of therapeutic hypothermia in the post-arrest patient is improved neurologic recovery

B Therapeutic hypothermia has a proven effect in reducing the rate of recurrent ventricular fibrillation B. Therapeutic hypothermia describes the practice of rapid cooling of patients to core temperatures of 32-42°C for 24-48 hours. This treatment approach has been shown to reduce the neurologic sequelae in the post-arrest patients. The treatment has also been suggested to improve the myocardial dysfunction associated with post-cardiac arrest patients. Therapeutic hypothermia has not been shown to reduce the recurrence of ventricular arrhythmia in post-arrest patients. Early coronary angiography and percutaneous coronary interventions have been demonstrated to prevent recurrent cardiac arrhythmias and sudden cardiac death.

A 16 year-old male presents with increasing pain and swelling of his right scrotum. The right testicle is extremely tender to palpation on examination. A Doppler ultrasound demonstrates decreased blood flow. Which of the following is the most appropriate intervention? A oral doxycycline B emergent surgery C incision and drainage

B emergent surgery

Based on current guidelines, which one of the following statements about intravenous fluid administration in patients with sepsis-related hypotension is correct? A Albumin is the fluid of choice, if available B 30 mL per kg of an intravenous crystalloid, such as saline, should be administered within the first three hours of presentation C A 1-L bolus of saline should be administered every hour until blood pressure normalizes D Administration of vasopressor therapy via peripheral administration is preferred over a central venous catheter

B 30 mL per kg of an intravenous crystalloid, such as saline, should be administered within the first three hours of presentation

You have been caring for a patient with sepsis and hypotension in the emergency department. Following the administration of the recommended amounts of intravenous fluid, the patient's mean arterial pressure remains below the target level of 65 mm Hg or greater. Which one of the following vasopressor therapies should be used first to increase the patient's mean arterial pressure? A Epinephrine B Norepinephrine C Vasopressin D Vasopressin plus epinephrine

B - Norepinephrine Vasopressor therapy is indicated if hypotension persists despite fluid administration. Norepinephrine is the first-line vasopressor agent for patients with septic shock if initial fluid resuscitation fails to restore mean arterial pressure to 65 mm Hg or greater.

A 32-year-old woman is noted to have a MAP < 65 mm Hg from suspected sepsis despite 4 L of IV normal saline. Which of the following is the best next step? A Use colloid (albumin) for the next bolus. B Initiate norepinephrine infusion. C Administer corticosteroid therapy. D Transfuse with fresh-frozen plasma. E Administer activated protein C.

B. A vasopressor agent such as norepinephrine (or dopamine) is the treatment of choice for hypotension that is unresponsive to intravenous saline infusion.

An unresponsive patient is brought to the ED by ambulance. You notice on his rhythm strip that he is in ventricular tachycardia with a heart rate of 210 beats/min and a blood pressure of 70/40 mmHg. The first step in treatment is to A. Administer IV adenosine B. DC cardiovert C. Administer IV lidocaine D. Apply overdrive pacer

B. DC cardiovert

Which of the following patients is most likely to have group A streptococcal infection? A An 11-month-old male infant with fever and red throat B An 8-year-old girl with fever and sore throat C A 27-year-old man with a temperature of 38.9°C (102°F), pharyngitis, and cough D A 52-year-old woman who complains of fever of 39.2°C (102.5°F) and sore throat

B. GABS is most common in patients younger than 15 years (although not in infants). McIsaac added age as a criterion because patients older than age 45 years have a much lower incidence of streptococcal pharyngitis.

A woman arrives in your ED with a human bite to her breast that occurred earlier in the day. There is a small puncture wound and no signs of cellulitis. A Identify the species, clean and immobilize the site, and administer antivenin. B Clean the bite site and treat with prophylactic antibiotics. C Clean the site, observe the animal, and watch for signs of secondary infection. D Clean the site and begin rabies prophylaxis with active and passive immunization. E Admit for radical surgical debridement in the operating room.

B. Human bites have high rates of infectivity. This wound does not appear to be infected. Nonetheless, the wound should be cleaned and a 3- to 5-day course of prophylactic antibiotics should be initiated. Human bites rarely lead to retained teeth, so a radiograph is not indicated. If this bite occurred on the hand or across a joint space, a radiograph should be performed. Tetanus toxoid should be given if indicated. TDaP has now been approved for use in patients over 65 years old.

An otherwise healthy young man presents with a 5-cm abscess on the lateral buttock. He is afebrile. The correct management includes all of the following except: A Pack the abscess and have the patient remove the packing himself within 24 hours and soak or bathe twice per day. B Treat with an oral first generation cephalosporin. C Incise with a scalpel and explore and open the cavity with a clamp. D Provide analgesia with oral ibuprofen and a ring of local anesthetic around the abscess.

B. In a healthy host, an abscess 5 cm or less with only minimal to moderate surrounding cellulitis usually does not require antibiotics. However, if antibiotics are prescribed, doxycycline, trimethoprim-sulfamethoxazole, or clindamycin is the correct choice. Long acting local anesthetic, such as bupivacaine, should be deposited in a ring around the abscess several minutes before incision and drainage. Packing or loop drainage is advised for abscesses that are more than a cm or so below the skin surface, as is commonly encountered in the buttocks, but it can be removed by the patient, with or without repacking. Soaking and scrubbing with soapy water is also recommended.

An 84-year-old woman is transferred from a nursing home with confusion, fever, and abnormal smelling urine. A lactate is noted to be 4.5 mmol/L. What are your initial priorities for this patient? A Order an intravenous pyelogram. B Rapid administration a 30 mL/kg fluid bolus and broad-spectrum antibiotics directed at urinary pathogens. C Initiate a workup for fictitious fever. D Consult a surgeon for possible appendicitis. E Discharge the patient back to the nursing home.

B. The patient has a severe urinary infection. The initial priorities in this patient are to rapidly administer a 30 mL/kg fluid bolus (to assist in perfusion and support BP) and broad-spectrum antibiotics directed at urinary pathogens.

A 48-year-old man is noted to have a 2-day history of sore throat, subjective fever at home, and no medical illnesses. He denies cough or nausea. On examination, his temperature is 38.3°C (101°F), and he has some tonsillar swelling but no exudate. He has bilateral enlarged and tender lymph nodes of the neck. The rapid streptococcal antigen test is negative. Which of the following is the best next step? A Oral clindamycin B Perform throat culture and treat based on results C Observation D Begin amantadine

B. This individual has a modified Centor score of 2 (history of fever, tender adenopathy, no cough, age >45). The rapid antigen test is negative, but a definitive culture should be performed for a Centor score of 2 or 3, and treatment should be based on culture results.

A 55-year-old man presents to the ED with complaints of a severe headache, diplopia, and vomiting. His blood pressure is 210/120 mm Hg upon arrival. Which of the following is the best next step? A Observe the blood pressure and recheck in 1 hour, and supportive measures for the headache and vomiting. B Obtain a head CT scan, give an antihypertensive such as nicardipine, and admit to the intensive care unit. C Give intravenous furosemide to decrease the blood pressure. D Give lorazepam to help the patient relax.

B. This man likely has hypertensive encephalopathy, which is a medical emergency. He has symptomatic HTN causing end-organ damage

Drug recommended in all stages of chronic heart failure

Beta blockers

A 76-year-old man presents to the emergency department 3 days after having a right-sided nasal packing placed for epistaxis with fever, headache, sore throat, myalgias, and abdominal pain. He is ill appearing and tachycardic. On examination, you notice a red, macular rash that involves his palms and soles as well as conjunctival injection. He states that it is not painful. What is the MOST LIKELY diagnosis? A Kawasaki's disease B Meningococcemia C Staphylococcal toxic shock syndrome D Toxic epidermal necrolysis

C Staphylococcal toxic shock syndrome

A 48 year-old male with a history of coronary artery disease and two myocardial infarctions complains of shortness of breath at rest and 2-pillow orthopnea. His oxygen saturation is 85% on room air. The patient denies any prior history of symptoms. The patient denies smoking. Results of a beta-natriuretic peptide (BNP) are elevated. What should be your next course of action for this patient? A Send him home on 20 mg furosemide (Lasix) p.o. every day and recheck in one week B Send him home on clarithromycin (Biaxin) 500 mg p.o. BID and recheck in 1 week C Admit to the hospital for workup of left ventricular dysfunction D Admit to the hospital for workup of pneumonia

C Admit to the hospital for workup of left ventricular dysfunction

Which of the following is most helpful in the diagnosis of a retropharyngeal abscess? A CBC with differential B fever and a muffled voice on examination C CT of the neck with contrast D history of a recent throat infection

C CT of the neck with contrast

Which of the following diagnostic tests should be ordered initially to evaluate for suspected deep venous thrombosis of the leg? A Venogram B Arteriogram C Duplex ultrasound D Impedance plethysmography

C Duplex ultrasound

An 87-year-old woman presents to the emergency department from a nursing home with fever and altered mentation. Vitals reveal a sinus tachycardia of 129 and blood pressure 79/55. She has an indwelling urinary catheter with purulent drainage. Laboratory abnormalities include a white blood cell count of 21.3 × 109/L, lactic acid level of 5.5, and glucose of 391. Which of the following is the BEST first step in management of this patient? A Acetaminophen per rectum B Intramuscular hydrocortisone C Intravenous access D Subcutaneous insulin

C Intravenous access

A 33-year-old woman with no significant medical history presents to the emergency department in January for fevers, chills, myalgias, malaise, headache, nasal congestion, and dry cough. She is 22 weeks pregnant with her first child. She states that she is a second-grade teacher and multiple children in her classroom have been ill with similar symptoms. Which of the following therapies should be offered to this patient? A Amantadine B Azithromycin C Oseltamivir D Supportive measures

C Oseltamivir

A 64-year-old man and his wife come to visit a family member in the hospital when the man suddenly collapses in the parking lot. His wife calls for help, and a nearby internal medicine resident immediately assesses the man, finding no pulse. The wife explains that her husband had been feeling vague chest discomfort the previous day. Which of the following is the most critical determinant of survival for this patient? A Time to cardiac catheterization B Time to epinephrine injection C Time to rhythm analysis and defibrillation D Time to chest compressions

C Time to rhythm analysis and defibrillation The leading cause of cardiac arrest in non-hospitalized individuals is ventricular arrhythmia - either ventricular tachycardia (VT) or ventricular fibrillation (VF). If the cardiac arrest is witnessed and a defibrillator can be accessed within 4-5 minutes (which it should be here, as they are at a hospital), then immediate defibrillation should be performed.

A 22 year-old male received a stab wound in the chest an hour ago. The diagnosis of pericardial tamponade is strongly supported by the presence of A pulmonary edema B wide pulse pressure. C distended neck veins. D an early diastolic murmur.

C distended neck veins.

A 75 year-old man with a long history of COPD presents with acute onset of worsening dyspnea, increased productive cough, and marked agitation. While in the emergency department he becomes lethargic and obtunded. His ABG's reveal a PaO2 40 mmHg, PaCO2 65 mmHg, and arterial pH 7.25. Which of the following is the most appropriate management at this point? A oxygen supplementation with a 100% non-rebreather mask B noninvasive positive pressure ventilation (NIPPV) C endotracheal intubation and mechanical ventilation D emergency tracheostomy

C endotracheal intubation and mechanical ventilation

Which of the following ECG findings is consistent with hyperkalemia? A prolonged QT interval B delta wave C peaked T waves D prominent U waves

C peaked T waves

A 28 year-old patient presents with complaint of chest pain for two days. The patient describes the pain as constant and sharp. It is worse with lying down, better with sitting up and leaning forward. Vital signs are BP 120/80, HR 80, regular, RR 14 and Temperature 100.1 degrees F. Which of the following would you expect to find on physical examination? A lower extremity edema B carotid bruit C pericardial friction rub D

C pericardial friction rub

Which of the following is an absolute contraindication to thrombolytic therapy in a patient with an acute ST segment elevation myocardial infarction? A history of severe hypertension presently controlled B current use of anticoagulation therapy C previous hemorrhagic stroke D active peptic ulcer disease

C previous hemorrhagic stroke

You are the first person on scene to a code blue in your hospital. You arrive to find an elderly woman who is unconscious, has a weak pulse, and does not appear to be breathing. Your first steps are: A Wait for the code cart to arrive and then intubate the patient. B Begin chest compressions and mouth-to-mouth resuscitation. C Attempt to remove any foreign body from the mouth and reposition the airway with chin lift or jaw thrust. D Begin bagging the patient immediately.

C Attempt to remove any foreign body from the mouth and reposition the airway with chin lift or jaw thrust. C. The most common cause of airway obstruction is the tongue and/or soft tissues of the upper airway. No other adjuncts may be necessary for initial management except relieving the obstruction with airway repositioning. This should certainly be the first step, and there is no need to wait for the code cart before performing this maneuver. There is no indication for chest compressions in a patient with palpable pulses. The patient will require BVM ventilation after airway repositioning and placement of an oral airway. If the patient is easy to ventilate, reversible causes of respiratory depression, such as a narcotic overdose, should be investigated and may eliminate the need for RSI.

A 30-year-old man presents with altered mental status, fever, and nuchal rigidity. You suspect bacterial meningitis. Which of the following is the appropriate order of your actions? A Head CT, lumbar puncture, blood cultures, steroids, antibiotics B Blood cultures, head CT, lumbar puncture, steroids, antibiotics C Blood cultures, steroids, antibiotics, head CT, lumbar puncture D Lumbar puncture, blood cultures, steroids, antibiotics, head CT E Head CT, blood cultures, steroids, antibiotics, lumbar puncture

C Blood cultures, steroids, antibiotics, head CT, lumbar puncture Neuroimaging is indicated in this patient prior to lumbar puncture given his altered mental status. Given the high suspicion for bacterial meningitis, antibiotic administration should not be delayed for the head CT. It is expected that one would obtain blood cultures and administer dexamethasone prior to the antibiotics in this case.

An 87-year-old woman presents with chest pain and shortness of breath. The 12-lead ECG shows a "sawtooth" pattern with a heart rate of 150 beats per minute. What is the most likely diagnosis? A AVNRT B VT C Atrial flutter D Atrial fibrillation with rapid ventricular rate

C. Classically, atrial flutter presents with a saw tooth pattern on ECG. The rate of 150 beats per minute denotes that it's likely a 2:1 conduction block.

A 55-year-old man presents to the ED complaining of right flank pain for the past 2 weeks. He has noted some gross hematuria and has been unable to eat anything secondary to nausea and vomiting. Which of the following is an indication for hospitalization? A Gross hematuria B Right flank pain C Nausea and vomiting despite antiemetics D Age greater than 50 years E Presence of a 6-mm stone

C. Hospitalization is required if the patient is unable to tolerate anything by mouth. Gross hematuria and flank pain are expected with nephrolithiasis.

A 55-year-old man has symptoms of worsening orthopnea, tachypnea, and rales on pulmonary examination. He has 2+ pitting edema in both lower extremities and jugular vein distention (JVD). Which of the following is the best description of this patient's disease process? A Right-sided HF B Left-sided HF C Biventricular HF D Acute respiratory distress syndrome

C. Left-sided HF will eventually progress to biventricular failure due to fluid overload of the right heart. This patient has evidence of both left and right HF: signs and symptoms of pulmonary edema (left), peripheral edema (right), and JVD (right).

A 62-year-old woman is sent to the ED from her primary physician's office with worsening HF. The patient has had CHF, previously controlled with oral digoxin and furosemide. Which of the following is the most likely reason for the exacerbation of her CHF? A Valvular dysfunction B Arrhythmia C Myocardial ischemia and infarction D Thyrotoxicosis

C. Myocardial ischemia and infarction is one of the most common precipitants of a CHF exacerbation (as well as noncompliance with medications). The other answer choices can also cause exacerbation of CHF but are less common.

A 35-year-old woman presents to the ED complaining of feeling lightheadedness. She noticed some vaginal bleeding earlier in the day. Her blood pressure is 85/53 mm Hg, heart rate is 130 beats per minute, and respiratory rate is 18 breaths per minute. Which of the following is the most appropriate next step in management? A Obtain a urine pregnancy test. B Obtain a serum quantitative beta human chorionic gonadotropin (β-hCG). C Obtain immediate IV access and begin fluid resuscitation. D Obtain stat OB/GYN consult.

C. Obtain IV access and begin fluid resuscitation. Investigating the possibility of pregnancy, specifically ectopic pregnancy, is critical. However, initial stabilization of the patient takes precedence. Hypotension must be treated emergently with fluids. Obtaining a consult early in the patient's course is important.

A 72-year-old man is brought to the ED by paramedics after fainting at the supermarket. His syncopal episode was witnessed by shoppers who stated the patient collapsed, hitting his head. The patient is currently alert and oriented and denies any persistent symptoms. His past medical history is significant for carotid stenosis, for which he takes aspirin and clopidogrel. What is most appropriate next step in the management of this patient? A Head CT scan B Order a carotid duplex ultrasound C Obtain an ECG D Chest radiograph

C. Obtain an ECG. This patient has a high probability for a cardiac cause of his syncope. Initial management includes placing the patient on a cardiac monitor and obtaining an ECG to monitor for dysrhythmias. A head CT scan should be performed after an ECG is obtained. It is most important to first rule out a dysrhythmia.

A 54-year-old man is seen at a rural ED with 1 hour of nausea and substernal chest pain radiating to his jaw. The ECG shows STEMI. PCI is not performed at this hospital and only performed at a hospital 2 1/2 hours away. Which of the following statements is most accurate? A Obtain a repeat ECG to assess if the patient truly has a STEMI. B Transfer the patient now for emergent PCI. C Administer thrombolytics, then transfer to PCI center. D Give aspirin, start heparin, and obtain a thallium stress test.

C. PCI is the treatment of choice when it can be performed rapidly by an experienced cardiologist. However, if a patient presents to a hospital that does not offer PCI and cannot be transferred to one within 120 minutes, administration of thrombolytics, preferably within 30 minutes of arrival, is indicated. The patient should then be transferred to the nearest PCI center.

A 45-year-old man is brought into the emergency center due to significant dehydration and weakness. His potassium level is noted to be 7.2 mEq/L. Which of the following statements is most accurate regarding his potassium level? A Hyperkalemia can usually be diagnosed by symptoms alone. B An ECG showing peaked T waves means the patient is stable and treatment can safely wait until laboratory results are obtained. C Hyperkalemia can mimic a myocardial infarction on the ECG. D Hyperkalemia is synonymous with kidney disease.

C. The ST-segment and T-wave changes of hyperkalemia may mimic the ECG appearance of myocardial infarction. The nonspecific symptoms typical of hyperkalemia are also often seen in patients with MI, particularly elderly patients. Peaked T waves indicate that the heart is significantly affected by hyperkalemia and the patient should not be considered stable. Many conditions and medications may cause hyperkalemia, not just renal failure.

A 66-year-old woman is noted to have acute severe pneumonia and has been treated with adequate fluid, antibiotics, and vasopressors. Which of the following is a sign that the patient is going into septic shock? A Urine output of >1 mL/kg/h B Mean arterial blood pressure of 80 mm Hg C Lactic acid level of 6 mmol/dL D Serum bicarbonate level of 22 mEq/L E Hematocrit 35%

C. The abnormal lactate level despite adequate fluid resuscitation is a sign that this patient is in septic shock. The patient's MAP of 80 mm Hg is adequate for perfusion. The other variables are not part of the new definition of septic shock.

A 35-year-old HIV-positive injection drug user presents, complaining of a hip abscess where he injects heroin. The temperature is 38.1°C, and there is a 10 × 10 cm circular area of erythema and induration on the lateral buttock without fluctuance. Which is the correct management? A Prescribe oral cephalexin for cellulitis and instruct the patient to return in 24 hours to assess whether an abscess has developed. B Attempt needle aspiration at the center of the infection, and if negative, cover with oral antibiotics. C Search for an abscess with bedside ultrasound and establish an IV in anticipation of a drainage procedure and admission for IV antibiotics. D Consult a surgeon immediately for suspected necrotizing SSTI.

C. This case is a classic presentation for a deep buttock or thigh abscess related to heroin injection. A septic hip or necrotizing infection should also be considered, although consultation for suspected NSTI is premature at this point. Nonpurulent cellulitis is very unlikely, and simply treating with antibiotics is incorrect management. These abscesses can be very large and may cause a low-grade fever. When there is no obvious fluctuance, imaging should be pursued with ultrasound, or occasionally CT, to confirm the diagnosis and guide drainage. Needle aspiration is reserved for small facial abscesses and has no proven diagnostic role. Given the fever, this patient will likely require IV antibiotics and admission, as well procedural sedation via IV

Your dog, who was immunized against rabies within the last year, bites your neighbor. A Identify the species, clean and immobilize the site, and administer antivenin. B Clean the bite site and treat with prophylactic antibiotics. C Clean the site, observe the animal, and watch for signs of secondary infection. D Clean the site and begin rabies prophylaxis with active and passive immunization. E Admit for radical surgical debridement in the operating room.

C. This is a low-risk bite. The dog is a house dog with a low risk of ever contracting rabies. You have it immunized every year and can observe it for 10 days. As always, clean the bite thoroughly and consider radiographs to be sure that no broken teeth are in the wound and that the bone has not been penetrated. Administer tetanus if indicated, and watch for secondary bacterial infection.

A 54-year-old woman presents to the ED requesting medication refills on her antihypertensive medications. She has been out of her medications for 2 weeks and cannot get an appointment with her private physician until next week. She normally takes atenolol and hydrochlorothiazide. Her blood pressure is 190/100 mm Hg. The patient has no complaints. She has been waiting for 4 hours and is in a hurry to get back to work. Which of the following is the most appropriate next step? A Change her medications to a CCB. B Admit to the intensive care unit and initiate intravenous nitroprusside. C Give her a prescription for her medications, instruct her to take them immediately and have her follow-up in 48 hours. D Counsel the patient on the dangers of her noncompliance, admit to the hospital, and begin the patient on intravenous labetalol.

C. This patient has hypertensive urgency. She has no symptoms related to her elevated blood pressure and no signs of end-organ damage. The patient should restart her medications and have her blood pressure reassessed in 48 hours.

What atbs cover MRSA?

Clindamycin, Vancomycin, Doxycycline, Bactrim

Which of the following is an expected finding in a patient with a diagnosis of an arterial embolism? A lower extremity edema B stasis dermatitis C palpable cord D pulselessness

D

Which of the following medication classes is the recommended treatment for patients who have an anterior wall myocardial infarction with poor left ventricular function? A Beta blockers B Calcium channel blockers C Potassium sparing diuretics D ACE inhibitors

D ACE inhibitors

Which of the following should be avoided in patients with heart failure? A Diuretics B Digoxin C Anticoagulants D Calcium channel blockers

D Calcium channel blockers

A 29 year-old male presents with complaint of substernal chest pain for 12 hours. The patient states that the pain radiates to his shoulders and is relieved with sitting forward. The patient admits to recent upper respiratory symptoms. On examination vital signs are BP 126/68, HR 86, RR 20, temp 100.3 degrees F. There is no JVD noted. Heart exam reveals regular rate and rhythm with no S3 or S4. There is a friction rub noted. Lungs are clear to auscultation. EKG shows diffuse ST segment elevation. What is the treatment of choice in this patient? A Pericardiocentesis B Nitroglycerin C Percutaneous coronary intervention D Indomethacin (Indocin)

D Indomethacin (Indocin)

A 23-year-old male being treated for acute bacterial prostatitis has been taking antibiotics for less than 24 hours. He presents to the emergency room today with acute urinary retention for 12 hours. Which of the following is the most appropriate next step? A Insert a Foley catheter B Initiate diuretic therapy C Schedule for cystoscopy D Insert a percutaneous suprapubic tube

D Insert a percutaneous suprapubic tube Inserting a percutaneous suprapubic tube is the treatment of choice in a patient with acute bacterial prostatitis who develops acute urinary retention. Rarely, acute urinary retention develops during an episode of acute prostatitis. In this setting, bladder drainage by suprapubic catheterization is the treatment of choice. Passage of a catheter through the inflamed urethra into the bladder in a patient with acute prostatitis risks septic shock or rupture of a potential abscess.

A 55-year-old male with history of hypertension and diabetes mellitus presents to the emergency department. The patient's wife states that the patient developed progressive irritability and confusion today after complaining of a headache. Physical examination reveals a BP of 230/130 mmHg and papilledema. Which of the following is the most accurate diagnosis in this patient? A Resistant hypertension B Hypertensive urgency C Hypertensive emergency D Malignant hypertension

D Malignant hypertension Malignant hypertension is characterized by diastolic reading greater than 140 mm Hg associated with papilledema and either encephalopathy or nephropathy.

A 25 year-old female presents to the emergency department after an episode of substernal chest pain with radiation to the middle of her back that came on suddenly and lasted for about four minutes this morning while in bed. She denies previous episodes. Examination is unremarkable, but she appears jittery. Toxicology screen is positive for cocaine. Which of the following medications is contraindicated in this patient? A Lorazepam (Ativan) B Diltiazem (Cardizem) C Nitroglycerin (Nitrostat) D Propanolol (Inderal)

D Propanolol (Inderal)

A 80-old-man with a history of diabetes, hypertension, and tobacco use presents to the emergency department with chest pain, nausea, vomiting, and shortness of breath. He takes a daily aspirin and insulin. His blood pressure is 84/65 with a heart rate of 74. He appears diaphoretic and distressed. You obtain this ECG with ST elevation in lead avF, II, and III. Which artery is occluded? A Left circumflex artery B Obtuse marginal artery C Proximal left anterior descending artery D Right coronary artery

D Right coronary artery

A 16-year-old patient presents with light headedness. You notice a systolic murmur on examination and an irregular cardiac shadow on his chest x-ray. You suspect he may have hypertrophic cardiomyopathy. Which of the following would increase the intensity of the murmur? A A fluid bolus of normal saline B Hand grip maneuver C Passive leg raise D Standing

D Standing

You are taking the history of a patient who is scheduled to have an internal cardiac defibrillator placed when the patient becomes unresponsive without a pulse. The nurse attaches a cardiac monitor and a chaotic rhythm with no discernible pattern is seen. What is the most likely diagnosis? A Atrial flutter B Ventricular tachycardia C Asystole D Ventricular fibrillation

D Ventricular fibrillation

A 55 year-old male presents with complaint of sudden ripping chest pain that radiates into the abdomen. On examination the patient is found to have diminished peripheral pulses and a diastolic murmur. EKG reveals left ventricular hypertrophy. Which of the following is the most likely diagnosis? A acute myocardial infarction B pulmonary embolism C acute pericarditis D aortic dissection

D aortic dissection

A 22-year-old baseball player comes to the ED complaining of 10 hours of severe right testicular pain. He denies a history of trauma. On examination, his right testis is diffusely tender and indurated, and the pain does not change with patient position. He has a cremasteric reflex on the right side. Which of the following is the best next step? A Continued observation B Oral antibiotics C Bed rest, ice to scrotum, and elevation of the scrotum D Emergent testicular ultrasound with color Doppler

D The clinical history is consistent with testicular torsion. The presence of a cremasteric reflex does not rule out the disease. Emergent testicular ultrasound is the next best step when the history and physical examination is concerning for testicular torsion.

Which of the following is/are the appropriate empiric antibiotics to administer to a 65-year-old man with suspected bacterial meningitis? A Vancomycin alone B Vancomycin and ceftriaxone C Vancomycin, ceftriaxone, and amoxicillin D Vancomycin, ceftriaxone, and ampicillin

D. All adults with suspected bacterial meningitis should receive a third-generation cephalosporin, and most institutions advocate for vancomycin to cover drug-resistant S. pneumoniae. Ampicillin is added because this patient is older than the age of 50.

A 55-year-old man collapses pulseless and with a wide bizarre-looking rhythm. A dialysis fistula is present in the right arm. In addition to standard ACLS therapies, which of the following is most appropriate for this patient? A 25 g of 50% dextrose, IV push. B Sodium bicarbonate, 50-mL IV push. C Begin immediate hemodialysis. D Calcium chloride, 20-mL slow intravenous push.

D. Calcium is the only agent with rapid and reliable enough onset to potentially help this patient. Bicarbonate might be appropriate, but its onset is slower than calcium and its effect is more disputed. Dialysis requires a hemodynamically stable patient. If the patient is resuscitated, dextrose and insulin will be important in the ongoing management of hyperkalemia.

Which of the following patients with pyelonephritis can be safely discharged home with close follow-up? A A 23-year-old woman in her second trimester of pregnancy B A 13-year-old woman who cannot tolerate her diet despite antiemetics C An 88-year-old man with urinary retention and dehydration D A 67-year-old woman with 3+ bacteria, a sulfa allergy, and a history of lupus E A 44-year-old woman with a kidney stone and hydroureter on CT scan

D. Despite a chronic medical condition, this patient may be safely discharged home. Because this patient has a sulfa allergy, TMP-SMX should not be administered. Other treatment options include quinolones, amoxicillin/clavulanate, and nitrofurantoin. All of the other patients should be admitted for treatment. All pregnant patients with pyelonephritis require admission. The 13-year-old and 88-year-old are not tolerating their diet and require intravenous hydration. The 44-year-old has a urinary obstruction with a UTI, which makes it a complicated UTI. These patients are at high risk for developing sepsis. For most admitted patients, urine cultures should be sent to guide antibiotic therapy.

An 18-year-old woman presents with fever and a sore throat. She meets criteria for treatments for GABS pharyngitis. Which medication would you prescribe (in addition to penicillin)? A Clindamycin B Erythromycin C Acyclovir D Dexamethasone E Methotrexate

D. Dexamethasone has been shown to decrease the amount of symptomatic time of pharyngitis and also to decrease pain. However, penicillin alone is sufficient, and no other medications are needed.

Which of the following statements is incorrect regarding treatment of hyperkalemia in patients with some renal function? A Administration of normal saline may hasten the excretion of potassium. B Administration of furosemide can hasten the excretion of potassium. C The combination of saline with a diuretic is often indicated because hyperkalemic patients are frequently dehydrated. D Patients with some preserved renal function do not need dialysis, even for severe hyperkalemia.

D. Dialysis is the definitive therapy for hyperkalemia. Patients who have residual kidney function can sometimes be managed without resorting to dialysis, but it should always be available for those who fail to respond quickly. Severe dehydration is fairly common in the scenario of the elderly patient "found down" for an unknown period. Such dehydration may promote potassium retention by the kidneys, as well as potassium shifts from acidosis and ischemia of vital organs.

A 65-year-old man with hypertension and benign prostatic hyperplasia (BPH) presents to the ED with urinary retention and a UTI on a catheterized urine analysis. He was evaluated by the urologist and is being discharged home with an in-dwelling Foley catheter and follow-up in the urology clinic in 1 week. Which of the following is the most appropriate antibiotic for this patient? A TMP-SMX bid for 3 days B Nitrofurantoin 100 mg for 14 days C Amoxicillin 100 mg tid for 14 days D Ciprofloxacin 500 mg bid for 14 days E Levofloxacin 250 mg qd for 3 days

D. Men with UTIs automatically fit into the "complicated" variety of UTIs. Therefore, the most appropriate therapy is ciprofloxacin for 14 days. With the exception of amoxicillin as monotherapy, all of the above choices are appropriate for treatment of certain types of UTIs. Complicated UTIs mandate 14 days of therapy with an appropriate antibiotic

A 40-year-old woman complains of a spider bite on her leg. It appears to be a necrotic 2 cm lesion on the right leg. What is the most likely diagnosis and etiologic organism? A Spider bite from a dermonecrotic spider species. B Impetigo from Group A Streptococcus. C Abscess from a polymicrobial mix of species including Streptococcus milleri. D Furuncle from methicillin-resistant S. aureus.

D. Necrotic spider bites are unusual, whereas spontaneous furuncles (superficial skin abscesses) are extremely common in emergency practice. Patients with furuncles often complain of a "spider bite." MRSA accounts for 50% to 60% of all SSTIs in US emergency departments and may be even more common in spontaneous furuncles. While most of these simple infections are cured by incision and drainage alone, if antibiotics are deemed necessary, MRSA coverage is a must, with either trimethoprim-sulfamethoxazole, doxycycline or clindamycin.

A 22-year-old baseball player comes into the ED complaining of 12 hours of intermittent chest pain and a pounding heartbeat. He denies a history of trauma. On examination, he is tachycardic. Which of the following is the best next step? A Synchronized cardioversion B Valsalva maneuver C Discharge home and follow-up within the next 48 hours D Obtain an ECG

D. One must characterize the rhythm before initiating treatment, which is why ECG is important to obtain. Answer C (discharge home) is not prudent since the patient has a tachycardia and it has not been diagnosed and may be dangerous. Therapy such as cardioversion or Valsalva requires a diagnosis first.

A 64-year-old woman is brought to the ED by her family for mental status changes. She has multiple sclerosis and self-catheterizes for urine. The family reports that over the past several days she has not been feeling well. They state that the patient vomited that day and was behaving bizarrely. Her vital signs are blood pressure of 83/38 mm Hg, heart rate of 135 beats per minute, respirations of 26 breaths per minute, and rectal temperature of 38.8°C (101.9°F). After a history and physical examination, which of the following is the most appropriate next step in management? A Obtain a urinalysis and culture B Start broad-spectrum antibiotics C Perform a lumbar puncture D Establish IV access and place the patient on a cardiac monitor E Discharge the patient after close follow-up is arranged

D. This woman may indeed have a UTI; however, her vital signs are unstable. The mainstay of treatment in emergency medicine is to first address the patient airway, breathing, and circulation (ABCs). This patient is hypotensive (BP 83/38 mm Hg). The first step in her management is placing an IV line and administering fluids. She should also be placed on a cardiac monitor to monitor her blood pressure, heart rate, and rhythm. Once her ABCs are addressed, laboratory studies should be obtained, including a urinalysis and culture. She should also receive broad-spectrum antibiotics and an antipyretic. This patient may need a lumbar puncture, but not until her ABCs are addressed. This patient requires admission to the hospital.

A 65-year-old man with history of diabetes presents to the Emergency Department stating he has had substernal chest pain and diaphoresis for 45 minutes. Which is the most important next step in management? A Sublingual nitroglycerin B Oxygen C Placement of defibrillator pads D Aspirin to chew E Morphine sulfate

D. While all of these interventions may be useful, aspirin significantly decreases mortality, with almost no downside in non-allergic patients, and it should be given immediately. Thus, aspirin is the first intervention.

In the initial evaluation of a patient with chest pain, which is the most important diagnostic test? A Treadmill stress test B PT C Chest x-ray D Troponin E ECG

E. The history of chest pain is important to obtain. The ECG is the crucial first diagnostic test in the evaluation of chest pain. Presence versus absence of ST elevation represents a major branch point in deciding treatment. Serum cardiac levels take time and sometimes are not elevated at the time of patient encounter.

A 58-year-old man complains of urgency, dysuria, lower back pain, and pain with ejaculation. A Torsion of the appendix testis B Testicular torsion C Epididymitis D Orchitis E Testicular tumor F Acute prostatitis

F. Acute prostatitis usually occurs in older patients. Urinary urgency, hesitancy, frequency, and perineal pain with ejaculation are common symptoms. The most common causative organism is Escherichia coli. Appropriate antibiotic choices include fluoroquinolones (ciprofloxacin, ofloxacin, and norfloxacin) as well as trimethoprim-sulfamethoxazole

An 87-year-old man is brought in by ambulance from a skilled nursing facility after being found unresponsive in bed with rapid, shallow breathing after 3 days of worsening cough. Paramedics report his room air saturation in the field was 67%. In the emergency department he is obtunded with sonorous respirations, labored breathing, and copious thick yellow secretions. His vital signs are: temperature 38.7°C, BP 90/58 mm Hg, P 118 bpm, RR 29 breaths per minute, and oxygen saturation 84% on a non-rebreather face mask. What is the immediate first step in the management of this patient? What special factors need to be considered?

First step: This patient needs immediate airway management and endotracheal intubation. Additional factors: As a critically ill patient, it is important to attempt to verify his code status before intubating. Considerations This patient has several concerning findings indicating he will need emergent airway management. He is hypoxic, tachypneic, and with his altered mental status, he may not able to protect his airway from aspiration of secretions or emesis. Because of his depressed level of consciousness and inability to protect his airway, he is not an appropriate candidate for noninvasive positive pressure ventilation (NIPPV) (such as bi-level positive airway pressure [BiPAP]). He likely has a pneumonia and/or aspiration event, but it is also important to consider that he may have had a separate preceding event such as a cerebral vascular accident or medication overdose which created the altered mental status before aspirating. His other vital signs indicate that he is probably septic and will need to be resuscitated after his airway is addressed.

A patient's EKG reveals widened P waves in lead II and large negative deflection of the P wave in lead V1. Which of the following is the most likely underlying cause for this?

Left atrial enlargement

Myocardial cell death caused by ischemia, evidenced by a rise and fall in cardiac biomarkers.

MYOCARDIAL INFARCTION

A 61-year-old man arrives at the emergency department (ED) complaining of chest pain. The pain began 45 minutes ago while watching television and is described as severe and pressure-like. It is substernal in location, radiates to the jaw and left shoulder, and is associated with shortness of breath. The patient has hyperlipidemia and hypertension and takes simvastatin and hydrochlorothiazide. His blood pressure is 160/110 mm Hg, pulse rate is 93 beats per minute, respiration rate is 22 breaths per minute, temperature is 37.1°C, and oxygen saturation is 97%. The patient appears anxious, is diaphoretic and vomited once. On auscultation, faint crackles are heard at both lung bases. The cardiac and abdominal examinations are unremarkable. What is the most likely diagnosis? What are the next diagnostic steps? What therapies should be instituted immediately?

Most likely diagnosis: Acute myocardial infarction. Next diagnostic steps: Place the patient on a cardiac monitor, establish IV access, and obtain an electrocardiogram (ECG) immediately. A chest x-ray and serum levels of cardiac markers should be obtained as soon as possible. Immediate therapies: Aspirin is the most important immediate therapy. Oxygen and sublingual nitroglycerin are also standard early therapies. Depending on the result of the ECG, emergency reperfusion therapy, such as a percutaneous coronary intervention (PCI), may be indicated. Intravenous beta-blockers, IV nitroglycerin, low-molecular-weight heparin, and additional antiplatelet agents, such as clopidogrel, might also be indicated.

A 63-year-old woman arrives in the emergency department (ED) in respiratory distress. The paramedics who transported her were not able to obtain any information about her past medical history but did bring her bag of medications, which included furosemide. On examination, her temperature is 37.5°C (99.5°F), blood pressure is 220/112 mm Hg, heart rate is 130 beats per minute, respiratory rate is 36 breaths per minute, and oxygen saturation is 93% on a non-rebreather mask. The patient's skin is cool, clammy, and diaphoretic. She is alert but can only answer yes-or-no questions because of dyspnea. She has jugular venous distention to the angle of the jaw, rales in both lung fields, and +2 pretibial edema bilaterally. Her heart sounds are regular but tachycardic, with an S3/S4 gallop. What is the most likely diagnosis? What is the most appropriate next step?

Most likely diagnosis: Congestive heart failure (CHF) and cardiogenic acute pulmonary edema. Most appropriate next step: Management of the ABCs (airway, breathing, circulation), preload and afterload reduction, and eventual diuresis.

A 55-year-old man is brought into the emergency department (ED) by his wife for altered mental status (AMS). She states that for the past day, he has been confused and unsteady when he walks. The patient has a history of hypertension (HTN) and hyperlipidemia. He complains of headache and blurry vision. On examination, he is alert and oriented to person only. On fundoscopy, the optic discs appear hyperemic and swollen, with a loss of sharp margins. His neurological examination is nonfocal, and he otherwise has a normal physical examination. The patient's vital signs are a blood pressure of 245/140 mm Hg, heart rate of 95 beats per minute, respiratory rate of 18 breaths per minute, and oxygen saturation of 98% on room air, and he is afebrile. What is the most likely diagnosis? What is the best management?

Most likely diagnosis: Hypertensive encephalopathy. Best management: Confirm the diagnosis by ruling out ischemic or hemorrhagic stroke, infection, and mass lesion. Lower blood pressure with intravenous (IV) medications and check for other evidence of end-organ damage.

A 30-year-old white man presents to the emergency department (ED) complaining of sudden onset of abdominal bloating and back pain. The patient states he was sleeping comfortably but then developed a sudden onset of severe, constant pain that radiates from his back to his abdomen and down toward his scrotum and caused him to be awakened. He is unable to find a comfortable position and feels best when he is able to get up to move around. He admits to having had occasional hematuria but denies ever having this type of pain in the past. He has no other significant medical problems. On physical examination the patient is diaphoretic and in moderate distress. His blood pressure is 128/76 mm Hg, heart rate is 106 beats per minute, temperature is 37.4°C (99.4°F), and respiratory rate is 28 breaths per minute. His cardiovascular examination reveals tachycardia without murmurs. Lung examination is clear to auscultation. Abdominal examination demonstrates normoactive bowel sounds, no abdominal distension, and significant costovertebral angle tenderness. A midstream voided urine specimen demonstrates gross hematuria. What is the most likely diagnosis? How would you confirm the diagnosis? What is the next step in treatment?

Most likely diagnosis: Nephrolithiasis. Confirmation of the diagnosis: Perform a urinalysis, complete blood count (CBC), serum chemistries, kidneys, ureters, bladder (KUB) radiograph, and intravenous pyelogram (IVP) or computed tomography (CT) scan of the abdomen. Next step in treatment: Start IV fluids and provide sufficient pain management for the patient before sending him for the appropriate imaging study. Strain all urine once the diagnosis of nephrolithiasis is suspected and perform stone analysis on any stone passed. **The ED provider must remain vigilant and rule out other acute abdominal etiologies that may be associated with a similar clinical presentation

A 73-year-old woman is brought to the emergency center from an assisted-living facility. The patient has a history of dementia, hypertension, and type II diabetes mellitus. By report, she has had chills and a productive cough for several days. In the past 24 hours she has become weaker and does not want to get out of bed. The physical examination reveals a thin, elderly woman who is somnolent but arousable. Her rectal temperature is 38.2°C (100.8°F), pulse rate is 118 beats per minute, blood pressure is 84/50 mm Hg, and respiratory rate is 22 breaths per minute. Her mucous membranes are dry. Her heart is tachycardic but regular. She has crackles at her right lung base with a scant wheeze. Her abdomen is soft and nontender. The extremities feel cool, and her pulses are rapid and thready. The patient is moving all extremities without focal deficits. What is the most likely diagnosis? What is the most likely etiology for this condition?

Most likely diagnosis: Sepsis. Most likely etiology: Pneumonia (Health-Care Associated).

A 45-year-old man presents to the Emergency Department (ED) complaining of left shoulder pain. Past history includes numerous skin abscesses, hepatitis C, and injection drug use. He injected black tar heroin into the left upper extremity 2 days ago. On examination the patient is in mild distress. There is a low-grade fever, the heart rate is 115 beats per minute, and blood pressure is 120/60 mm Hg. The dorsum of the upper arm is erythematous, indurated and tender. There is no obvious area of fluctuance. Edema extends to the shoulder and pectoralis region of the trunk. What is the most likely diagnosis? What are the next diagnostic and treatment steps?

Most likely diagnosis: Soft tissue abscess from injection drug use. However, necrotizing soft tissue infection (NSTI) is a distinct possibility, and the differential diagnosis also includes cellulitis and septic shoulder joint. Next steps: Establish IV access. IV antibiotics are generally indicated when a SSTI produces a fever. Establish a definitive diagnosis as quickly as possible, beginning with a careful search for a pus pocket. If an abscess is found, it must be drained. If not, NSTI remains a possibility and immediate surgical exploration is indicated. Search for signs of sepsis, and if present begin early goal directed therapy.

A 13-year-old adolescent boy presents to the emergency department with a chief complaint of sore throat and fever for 2 days. He reports that his younger sister has been ill for the past week with "the same thing." The patient has pain with swallowing but no change in voice, drooling, or neck stiffness. He denies any recent history of cough, rash, nausea, vomiting, or diarrhea. He denies any recent travel and has completed the full series of childhood immunizations. He has no other medical problems, takes no medications, and has no allergies. On examination, the patient has a temperature of 38.5°C (101.3°F), a heart rate of 104 beats per minute, a blood pressure of 118/64 mm Hg, a respiratory rate of 18 breaths per minute, and an oxygen saturation of 99% on room air. His posterior oropharynx reveals erythema with tonsillar exudates without uvular deviation or significant tonsillar swelling. Neck examination is supple without tenderness of the anterior lymph nodes. Chest and cardiovascular examinations are unremarkable. His abdomen is soft and nontender with normal bowel sounds and no hepatosplenomegaly. Skin is without rash. hat is the most likely diagnosis? What are the dangerous causes of sore throat you don't want to miss? What is your diagnostic plan? What is your therapeutic plan?

Most likely diagnosis: Streptococcal pharyngitis. Dangerous causes of sore throat: Epiglottitis, peritonsillar abscess, retropharyngeal abscess, and Ludwig's angina. Diagnostic plan: Use Centor criteria to determine probability of bacterial pharyngitis and rapid antigen testing when appropriate. Therapeutic plan: Evaluate the patient for need of antibiotics versus supportive care.

A 25-year-old man presents to the emergency department (ED) with palpitations and lightheadedness. These symptoms started acutely about 1 hour prior to arrival while he was watching television. The patient does not have any chest pain or shortness of breath. He also denies any recent fever, upper respiratory symptoms, and hemoptysis. He does not have any significant past medical history or family history. He is not taking any medications, does not smoke, and has never used any illicit drugs. On examination, his temperature is 98.2°F, blood pressure is 88/46 mm Hg, heart rate is 186 beats per minute, respiratory rate is 22 breaths per minute, and oxygen saturation is 97% on room air. He is mildly anxious but otherwise in no acute distress. He does not have any jugular venous distention. His lungs are clear to auscultation, and his heart sounds are regular without any murmurs, rubs, or gallops. There is no lower extremity edema, and peripheral pulses are equal in all four extremities. The cardiac monitor reveals a regular rhythm with narrow-QRS complexes at a rate of 180 to190 beats per minute. What is the most likely diagnosis? What is the most appropriate next step?

Most likely diagnosis: Supraventricular tachycardia (SVT). Most appropriate next step: Obtain IV access and a 12-lead ECG. Prepare for synchronized cardioversion of this unstable patient with a tachyarrhythmia. The physical examination will initially focus on assessing the patient's stability and adequacy of the airway, breathing, and circulation (ABCs). Any evidence of hypotension, pulmonary edema, acutely altered mental status, or ischemic chest pain indicates that the patient is unstable and that treatment must be initiated immediately (see treatment section later in chapter). Once the patient is stabilized, a complete head-to-toe examination can be performed.

A 17-year-old adolescent male arrives at the emergency department (ED) after he developed an acute onset of severe right testicular pain about 4 hours ago while at soccer practice. The patient does not recall any recent trauma to the area and denies any fever, dysuria, or penile discharge. Although he has nausea, he does not have any abdominal pain or vomiting. On examination, his temperature is 99.5°F, blood pressure is 138/84 mm Hg, heart rate is 104 beats per minute, and respiratory rate is 22 breaths per minute. He is in acute distress due to pain. His abdomen is benign. On visual inspection, he has right scrotal erythema and swelling, although there are no penile lesions or discharge. Because his scrotum is so diffusely tender, it is difficult to examine it more closely. However, there is no testicular rise when his inner thigh is stroked. His urinalysis shows 3-5 white blood cells (WBCs)/high power field (hpf). What is the most likely diagnosis? What is the next diagnostic step?

Most likely diagnosis: Testicular torsion. Next diagnostic step: Urological consultation. Manual detorsion can be attempted while awaiting the consultant.

A 15-year-old adolescent boy was cleaning some items in the shed in his backyard in the middle of the afternoon when he saw a bat in middle of the shed. The bat bit the boy on his dominant hand, after which the teenager ran into the house. His parents brought the boy to the emergency department. His vital signs on arrival were a blood pressure of 115/70 mm Hg, heart rate of 105 beats per minute, respiratory rate of 14 breaths per minute, pulse oximetry of 99% on room air, and temperature of 37.1°C (98.9°F). Inspection of the wound shows deep bite marks with a laceration close to the proximal interphalangeal joint. The bat escaped after the boy was bitten and was not found. What is the most likely diagnosis? What is the next step in treatment?

Most likely diagnosis: Unprovoked attack by a rabies-infected bat. Best initial treatment: Notify animal control to locate the animal, then clean the wound and administer both passive and active rabies immunization to the patient. Administer tetanus toxoid if not received within the last 5 years.

A 24-year-old woman presents to the emergency department (ED) with complaints of flank pain and fever for the last 1-2 days. She describes feeling pain with urination over the previous week. She is currently feeling febrile and nauseated but has not vomited. The pain in her right flank is a dull, constant, nonradiating ache that she rates as 5/10 for pain. She took 600 mg of ibuprofen last night to help her sleep, but this morning the pain persisted, so she came into the ED for evaluation. She reports that she is sexually active and that her last menstrual period was 1 week ago. She denies any vaginal discharge or abdominal pain. Her vital signs include a temperature of 38.3°C (101°F), heart rate of 112 beats per minute, respiratory rate of 15 breaths per minute, and blood pressure of 119/68 mm Hg. Her examination is significant for tenderness to palpation on her right costovertebral angle (CVA). What is the most likely diagnosis? What is the best treatment?

Most likely diagnosis: Urinary tract infection (UTI) complicated by pyelonephritis. This should be confirmed with a urinalysis. Treatment: Antibiotics, hydration, analgesia, antipyretics, and exclusion of other pathology. Uncomplicated pyelonephritis can be treated in the outpatient setting, provided the patient can tolerate oral medications, has mild symptoms, gets good follow-up, and is not pregnant. TMP-SMX, amoxicillin/clavulanate, or a fluoroquinolone antibiotic should be prescribed for 10-14 days.

Treatment for unstable a fib

Patients who are hemodynamically unstable should get immediate electrical cardioversion to restore sinus rhythm.

Treatment for stable AF

Rate control and/or rhythm control, with or without anticoagulation Rate control drugs: CC blockers B blockers Digoxin Amiodarone

A 72 year-old male presents to the emergency department with crushing chest pain, dyspnea and palpitations for 2 hours in duration. Enzymes are pending and he has been given aspirin and sublingual nitroglycerin. He is rushed to the catheterization lab where they find a totally occluded distal right coronary artery. Which of the following electrocardiogram (ECG) findings supports the diagnosis? A Q waves in leads I, aVL, V5-V6 B ST segment elevation in leads II, III, aVF C Hyperacute T waves in leads I, aVL D Flipped T waves with repolarization changes in leads V1-V4

ST segment elevation in leads II, III, aVF, represents an acute process in the right coronary artery.

A 37-year-old man with human immunodeficiency virus presents to the emergency department with two days of fever and a new-onset seizure tonight. After completing a thorough physical and neurologic examination, what is the NEXT BEST step in management? A Head CT B Intravenous lorazepam C Lumbar puncture D Measure CD4+ T-cell counts

The answer is A. (Chapter 155) Initial emergency department evaluation of human immunodeficiency virus (HIV) patients presenting with new neurologic symptoms or deficits should include non-contrast-enhanced CT is the first neuroimaging study to be obtained. Contrast-enhanced CT or MRI may be obtained next if the non-contrast-enhanced CT is equivocal or there is concern for malignancy or abscess. The patient should undergo lumbar puncture for further infectious evaluation if head imaging is unremarkable, but a space-occupying lesion should be ruled out with CT before this is attempted. This patient may require intravenous lorazepam if he has another seizure but does not currently need it. Determining his CD4+ T cell count may assist in guiding treatment if an infectious cause is found but should not delay head CT.

Which of the following is a characteristic feature of staphylococcal toxic shock syndrome (TSS) erythroderma? A Involves the palms B Painful C Petechial D Spares the mucosa

The answer is A. (Chapter 249) Toxic shock syndrome erythroderma is characterized as a painless, diffuse, red, macular rash resembling "sunburn" and involves the palms and soles. Mucosal involvement can include conjunctival and scleral hemorrhage as well as vaginal, cervical, or oropharyngeal hyperemia ("strawberry tongue").

A patient presents after an automobile versus pedestrian accident and is found to have an open ankle fracture dislocation with diminished distal pulses. The patient is prepped for an emergent reduction under procedural sedation. He is placed on a cardiac monitor with capnography and pulse oximetry. His vital signs are within normal limits, and his oxygen saturation is 100%. Airway equipment is available at the bedside. Which of the following describes the NEXT step? A Administer fentanyl IV B Administer hydromorphone IV C Sedate with etomidate IV D Remove supplemental oxygen as it can delay the recognition of respiratory suppression

The answer is A. (Chapter 37) It is appropriate to administer a short-acting analgesic agent prior to the procedure when time permits. Shorter-acting agents (such as fentanyl and morphine) are preferred to drugs with a longer duration of action (such as hydromorphone) to minimize postprocedure respiratory depression. Begin procedural sedation after the last dose of analgesic has reached its peak effect (3-5 minutes for IV morphine and 2-3 minutes for IV fentanyl). Despite its potential to delay the recognition of respiratory suppression, supplemental oxygen reduces the incidence of hypoxemia and has no adverse clinical effects and should therefore be utilized in procedural sedation.

Which of the following correctly lists the mortality risk associated with syncope etiology from highest to lowest? A Cardiac, neurologic, unknown, vasovagal B Cardiac, neurologic, vasovagal, unknown C Neurologic, cardiac, unknown, vasovagal D Unknown, cardiac, neurologic, vasovagal

The answer is A. (Chapter 52) The Framingham study showed that cardiac syncope doubled the risk of death, neurologic syncope increased the risk of death by 50%, and unknown cause of syncope increased the risk of death by 30%. Vasovagal syncope carried no increase in risk of death when compared to the general population.

A 50-year-old man with chronic hypertension presents with chest pain described as sudden in onset and radiating to his back. You are worried about aortic dissection. Which of the following tests can BEST rule out an aortic dissection? A Chest CT B Chest radiograph (x-ray) C D-dimer D ECG

The answer is A. (Chapter 59) Chest CT is the diagnostic test of choice and can accurately rule out aortic dissection. In addition to diagnosing dissection, it can diagnose other pathology such as intramural hematoma or penetrating atherosclerotic ulcer.

Which of the following exposures mandates rabies postexposure prophylaxis? A Awakening in a room with a bat present B Cleaning a stray dog with unknown origins C Contact with rabid dog blood on intact skin D Petting a cat with raccoon saliva on its coat

The answer is A. Any direct contact between a human and a bat should be evaluated for a rabies exposure. Seeing a bat does not constitute an exposure. Postexposure prophylaxis is recommended for persons who were in the same room as a bat and who were unaware if a bite or direct contact had occurred (e.g., a sleeping person awakens to find a bat in the room, or an adult witnesses a bat in a room with an unattended child, mentally disabled person, or intoxicated person). A nonbite exposure is contamination of scratches, abrasions, open wounds, or mucous membranes with saliva or brain tissue from a rabid animal. For example, animal licks to nonintact skin have transmitted rabies. Nonbite exposures from animals very rarely cause rabies. If the material containing the virus is dry, the virus can be considered noninfectious. Petting a rabid animal or contact with blood, urine, or feces of a rabid animal does not constitute an exposure and is not an indication for prophylaxis.

Which of the following risk factors is more commonly associated with staphylococcal toxic shock syndrome, as opposed to streptococcal toxic shock syndrome? A Influenza B Retained foreign body (tampon, nasal packing) C Surgery D Varicella

The answer is B. The vagina and nares are known sites of Staphylococcus aureus colonization. Retained foreign bodies such as tampons, female barrier contraceptives, and nasal packing material are risk factors for developing staphylococcal toxic shock syndrome (TSS). Influenza, surgery, and varicella are associated with both staphylococcal and streptococcal TSS.

A 56-year-old man with diabetes presents with pain and swelling in his legs. His symptoms improve with hanging his foot over his bed. On examination he is noted to have an irregular heart rhythm with pallor and diminished pulses to his leg. He is diagnosed with acute embolic limb ischemia from atrial fibrillation. Atrial fibrillation is the most common cause of acute embolic limb ischemia, but what is the second most common cause? A Atrial myxomas B Mural thrombus C Prosthetic heart valve D Vegetations from valve leaflets

The answer is B. (Acute limb ischemia due to embolism is less common than occlusion from thrombosis. This is in part due to the decline in rheumatic heart disease and the improvement in the management of patients with atrial fibrillation. Even with this improvement, atrial fibrillation is still the leading cause of embolic ischemic disease. The second most common source is from a mural thrombus that forms in the ventricle after a recent myocardial infarction (MI). This accounts for approximately 20% of all limb emboli. Heart valves, myxomas, and vegetations are rare causes of embolic disease.

A 22-year-old woman presents for a second visit to the emergency department 3 days after being evaluated for dysuria. She has persistent dysuria despite being prescribed nitrofurantoin. Review of her urinalysis from 3 days ago reveals multiple WBCs, a negative pregnancy test, and a negative culture. Which of the following may be a complication if this infection is not treated appropriately? A Meningitis B Pelvic inflammatory disease C Pustular skin lesions D Septic arthritis

The answer is B. (Chapter 153) Urethral chlamydial infection should be considered in the differential diagnosis of sterile pyuria for men and women. Complication of untreated infection may include pelvic inflammatory disease. Other complications may include ectopic pregnancy and infertility. Pustular skin lesions, meningitis, and septic arthritis are more commonly associated with gonococcal infections.

A 47-year-old woman with human immunodeficiency virus infection presents to the emergency department with 2 days of headache, increasing left-sided weakness, and new-onset seizure tonight. Contrast-enhanced CT scan shows multiple ring-enhancing lesions with surrounding areas of edema. Which of the following statements regarding this condition is TRUE? A Serologic tests are useful in making or excluding the diagnosis. B Symptoms may also include fever and altered mental status. C This condition commonly occurs in patients with normal CD4+ cell counts. D This patient can be discharged with oral sulfamethoxazole-trimethoprim and dexamethasone.

The answer is B. (Chapter 155) Toxoplasmosis most commonly occurs in patients with CD4+ cell counts <100 cells/mm3. Symptoms may include headache, fever, focal neurologic deficits, altered mental status, and seizures. Serologic tests are not useful in making or excluding the diagnosis because antibodies to Toxoplasma gondii are prevalent in the general population. The presence of antibodies to T. gondii in the CSF is helpful, although there is a high rate of false-negative results. On unenhanced CT scan, toxoplasmosis typically appears as multiple subcortical lesions with a predilection for the basal ganglia. Contrast-enhanced CT scan typically shows multiple ring-enhancing lesions with surrounding areas of edema.

Question 23 of 200 A 22-year-old woman presents to the emergency department with complaints of fever, headache, arthralgias, and right-sided facial weakness. Three weeks ago, after returning from a trip to the eastern coast of the United States, she was ill with 3 days of fever and myalgias, which resolved spontaneously. On examination, she is ill-appearing with a right facial droop that involves the forehead. There are no other neurological deficits on examination. What is the MOST APPROPRIATE treatment for this condition? A Azithromycin B Ceftriaxone C Doxycycline D Streptomycin

The answer is B. (Chapter 161) Lyme disease is the most common vector-borne zoonotic infection in the United States, with majority of cases reported in the northeast and upper midwestern states. The infecting organism is Borrelia. burgdorferi, a spirochete, transmitted by the Ixodes deer tick, also known as the black-legged tick. Lyme disease has three clinical stages. The first stage is local and classically characterized by a rash, erythema migrans, with other nonspecific symptoms of fever, chills, fatigue, myalgias, arthralgias, and lymphadenopathy. The second stage, early disseminated disease, is characterized by fever, adenopathy, neuropathies, cardiac abnormalities, arthritic complaints, and skin lesions. One of the most common neurologic symptoms in the secondary stage of illness is unilateral or bilateral facial nerve palsy. The third stage, late disseminated stage, occurs months to years after the initial infection and is characterized by chronic arthritis, myocarditis, subacute encephalopathy, axonal polyneuropathy, and leukoencephalopathy. The preferred treatment of Lyme disease is with doxycycline. However, severe disease or neurologic symptoms require ceftriaxone therapy.

Emergency medical services (EMS) responds to a call for a 75-year-old man with no known past medical history who is prescribed no medications. He complains of chest pain and mild shortness of breath. The patient's vital signs are BP 75/45, HR 102, RR 22, SpO2 97% on RA, and T 37°C. The patient has clear lung sounds and faint equal radial and femoral pulses. EMS established an IV and obtained an ECG that demonstrates ST elevation in the inferior leads with ST depression in leads I and aVL. You are contacted to provide medical command. Which of the following is the BEST prehospital advice for this patient? A Administer 324 mg chewable aspirin, a 500-mL IV normal saline bolus, and transfer to the closest emergency department 5 minutes away that does not have percutaneous coronary intervention (PCI) B Administer 324 mg chewable aspirin, a 500-mL IV normal saline bolus, and transfer to an emergency department 15 minutes away that does have PCI and cardiac revascularization capability C Administer 324 mg of chewable aspirin, 500-mL IV normal saline bolus, 5 mg IV metoprolol, and transfer to the closest emergency department 5 minutes away that does not have PCI D Administer 324 mg chewable aspirin, 500-mL IV normal saline bolus, 5 mg IV metoprolol, and transfer to an emergency department 15 minutes away that does have PCI and cardiac revascularization capability

The answer is B. (Chapter 50) The patient presents with cardiogenic shock due to ischemia. The most important intervention for ischemic-related cardiogenic shock is emergent revascularization. Stabilization in the emergency department is temporizing, and the ultimate goal is definitive therapy as fast as possible. In the prehospital setting, Emergency Medical Services should direct any suspected cardiogenic shock to a facility that has 24-hour emergency cardiac revascularization capability. Therefore, answers A and C are incorrect because the 10 minutes saved in transport do not make up for the patient benefit of presenting to the facility that can manage his specific disease. Answer D is incorrect because metoprolol is contraindicated in a patient presenting with cardiogenic shock.

Heart failure has a poor prognosis with approximately 50% of patients diagnosed dying within what period of time? A 2 years B 5 years C 10 years D 20 years

The answer is B. (Chapter 53) According to the American Heart Association 2014 Heart Disease statistics, survival after heart failure diagnosis has improved over time. However, the death rate remains high; approximately 50% of people diagnosed with heart failure will die within 5 years.

You are examining a 74-year-old man in the emergency department who tells you he has a cardiac valve that was replaced several years ago. On chest auscultation you hear a loud, clicking metallic sound associated with S1. Which of the following is TRUE regarding mechanical valves when compared to bioprosthetic valves? A Mechanical valves are more frequently rejected by the host immune system. B Mechanical valves carry an increased risk of stroke. C Mechanical valves fail more frequently. D Mechanical valves require surgical replacement more frequently.

The answer is B. (Chapter 54) Mechanical valves are more durable with lower failure rates, but they do carry a higher risk for thromboembolic complications. Antiplatelet therapy is recommended for all patients with prosthetic valves; however, for patients with mechanical valves lifelong anticoagulation is recommended. The risk of valve thrombosis or thromboembolism is about 8% and falls to 1% to 2% per year with anticoagulation

An 88-year-old man presents to the emergency department with severe abdominal pain. He reports difficulty urinating over the past few weeks and has not been able to void since last night. What is MOST COMMON etiology of acute urinary retention in this demographic? A Adverse medication effect B Benign prostatic hypertrophy C Neurogenic bladder D Tumor

The answer is B. (Chapter 92) Although benign prostatic hypertrophy (BPH) is the most common reason for acute urinary retention in the emergency department, it is important to tease out possible factors in the history and physical examination which could point to alternative etiologies. Relevant historical factors could include a history of prostatism, nocturia, or terminal dribbling. Incontinence may be associated with BPH secondary to overflow, but symptoms like urgency, frequency, and hesitancy may occur with infectious causes as well. Infectious sources may cause accompanying fever and chills. If the patient has a history of trauma, bladder calculi, surgery, or radiation to the prostate or bladder, these facts may steer the physician toward alternative diagnostic possibilities. Common pharmacologic causes of urinary retention include anticholinergics, opiates, anesthetics, and decongestants.

A healthy 33-year-old woman is diagnosed in the emergency department with a 2-mm nonobstructive stone at the ureteropelvic junction. Her urine analysis shows 57 red blood cells (RBCs) but is otherwise normal. Her pain is controlled with 15 mg of Toradol. Which of the following are the MOST APPROPRIATE discharge instructions? A Pain medication as directed, strain urine and drink plenty of fluids. Follow-up with urology electively. B Pain medication as directed, strain urine and follow up with urology or primary care within 1 week. Return for high fevers, severe pain unresponsive to analgesics, or persistent nausea and vomiting. C Pain medication as directed, strain urine, urgent next day urology follow-up D Pain medication as directed, strain urine, urgent next day urology follow-up, return to emergency department if stone has not passed in 2 days.

The answer is B. (Chapter 94) Discharging patients with nephrolithiasis is appropriate if the stone is small enough to likely pass spontaneously, pain is controlled by oral analgesics and there are no signs of significant infection. After discharge, patients can strain their urine in order to capture a stone for potential analysis. Prescriptions for oral analgesics and medical expulsive therapy should be given. Return precautions should include severe unrelenting pain, fevers, and persistent vomiting. Urology or primary care follow-up within 1 week is appropriate. Patient education is important. They should be aware that stone passage rates are variable depending on stone size and the location. Larger kidney stones may take up to 30 days to pass.

An 85-year-old man with a history of stage 5 chronic kidney disease, metastatic prostate cancer, and history of exploratory laparotomy for a gunshot wound presents with acute sharp left lower quadrant abdominal pain. He does not take any medications. Vital signs are BP 130/70, HR 80, RR 14, SpO2 95% on RA, T 37.9°C. Examination is notable for tenderness to palpation to the left lower quadrant. Laboratory studies show a glomerular filtration rate (GFR) of <15 mL/min/1.73 m2. Which of the following is the MOST APPROPRIATE initial pharmacotherapy for this patient? A Fentanyl 100 mcg nasal spray in each nostril B Hydromorphone 0.0075 mg/kg IV C Ibuprofen 800 mg orally D Morphine patient-controlled analgesia

The answer is B. In patients with renal failure, hydromorphone and fentanyl are the preferred opioids.

A 90-year-old woman presents to the emergency department from a nursing home with fever and altered mentation. Vitals reveal a sinus tachycardia of 128, respiratory rate of 28, and blood pressure 85/55. Laboratory abnormalities include a white blood cell count of 19.8 × 109/L and lactic acid of 5.5. Which of the following etiologies MOST LIKELY accounts for her presentation? A Cellulitis B Meningitis C Pneumonia D Pyelonephritis

The answer is C. (Chapter 151) Sepsis is a clinical diagnosis in the emergency department when the practitioner has suspicion or confirmation of infection, systemic inflammation, and new organ dysfunction or tissue hypoperfusion. The most common etiology of sepsis is acute bacterial pneumonia and the most common causative organisms are Streptococcus pneumoniae, Staphylococcus aureus, gram-negative bacilli, and Legionella pneumophila. Cellulitis and pyelonephritis are also common causes of sepsis but less common than pneumonia. Meningitis is a rare but devastating cause of sepsis.

A patient presents to the emergency department with a blood pressure of 235/140. He states that his blood pressure has never been this high before. Upon further questioning, he admits to running out of his antihypertensive medication 2 days ago. Which medication was he MOST LIKELY taking? A Amlodipine B Clevidipine C Clonidine D Enalapril

The answer is C. (Chapter 57) Clonidine is an α2-agonist. Abrupt cessation can cause significant rebound hypertension.

A young boy presents with trauma to his penis from a zipper being stuck on it. What are the BEST potential treatments for this problem? A Analgesics, application of ice pack, transfer for urologic evaluation B Injection of lidocaine 1% plus epinephrine, use of a Kelly clamp to remove entrapped tissue C Local anesthetic, application of mineral oil or surgical lubricant, use of a wire cutter D Oral narcotic analgesia, use of a scalpel to free the affected tissue from entrapment

The answer is C. (Chapter 93) Zipper entrapment of the penis or scrotum is the most common genital injury in male pediatric patients. Local anesthetic can be used, and then it is recommended to attempt application of a nontoxic lubricant to ease removal of the zipper. If this alone is not effective, a wire cutter can be used to cut the median bar of the zipper, which is easily accessible in the case of the image provided. If only the zipper teeth are involved, a cut can be made to the cloth in between the teeth to release the zipper. Use of a scalpel on the entrapped tissue should not be necessary. Lidocaine with epinephrine is generally avoided in penile injuries.

A 53-year-old man presents to the emergency department with chest pain. His blood pressure is 250/170. Due to his severe hypertension and concerning description of the pain, he is immediately started on an antihypertensive medication. Five hours later the admitting team mentions that they are concerned about cyanide toxicity if the patient stays on the medication too long. Which medication was started? A Amlodipine B Esmolol C Nesiritide D Nitroprusside

The answer is D

A 3-year-old boy weighing 16 kg presents with a crush injury to the left upper extremity and is given a rapid bolus of 50 mcg IV fentanyl for acute traumatic pain. Moments later he is noted to be apneic and cannot be effectively ventilated. Which of the following is the definitive step in management of this patient? A Administer IV flumazenil B Administer IV naloxone C Insert oral airway and continue bag valve mask ventilation D Perform rapid sequence intubation (RSI)

The answer is D. (Chapter 37) This child developed rigid chest syndrome, a rare complication characterized by spasm of the respiratory muscles leading to respiratory depression or apnea. Rigid chest syndrome may not be reversible with opioid receptor antagonists such as naloxone, and bag valve mask ventilation may not provide adequate ventilation. Intubation with rapid-sequence induction and pharmacologic paralysis is usually required to ventilate a patient in this situation.

A 54-year-old woman is brought to the emergency department by Emergency Medical Services for respiratory distress. The patient's vital signs are HR 107, BP 170/95, RR 32, SpO2 86%, and T 37.2°C. Her examination is remarkable for tachypnea, rales bilaterally on auscultation, and jugular venous distension. She is sitting up on the stretcher, pulling off her cardiac monitor leads, and not answering questions or following commands. IV access has been obtained. What is the BEST NEXT step? A Obtain a stat ECG and chest x-ray B Place the patient on a non-rebreather, administer IV furosemide, and start a nitroglycerin infusion C Place the patient on bilevel positive airway pressure (BiPAP) D Prepare for emergent intubation

The answer is D. (Chapter 53) The patient is in severe respiratory distress secondary to a heart failure exacerbation. The patient is not following commands, agitated, hypoxic, and uncooperative with altered mental status. The correct course of action is emergent intubation. BiPAP is contraindicated in patients who are unable to protect their airway due to poor mental status. Placing a non-rebreather is also not going to protect the patient's airway, and given her respiratory status, she will benefit from positive pressure ventilation to correct her hypoxia and work of breathing. Administration of furosemide and nitroglycerin may be helpful in her management but are not the first priority in a patient with an unprotected airway.

What BEST characterizes a complicated urinary tract infection (UTI)? A UTI in adults over the age of 65 B UTI with significant dysuria and frequency C UTI with fever D UTI in a patient with abnormal anatomy or function of GU tract with comorbidities that increase the risk of adverse outcomes

The answer is D. (Chapter 91) A complicated urinary tract infection (UTI) is defined as an infection of genitourinary (GU) tract in a patient with abnormal anatomy or physiology of the renal system and with comorbidities that place the patient at risk of adverse outcomes. Complicated UTI is not defined by clinical symptoms. Advanced age is a risk factor for complicated UTI but does not define it.

You are evaluating a 67-year-old patient with no known medical history who reports a fever, shaking chills, and scrotal pain and redness that is spreading rapidly. Inspection of the area reveals the following findings (its a photo of a really red scrotum). His fingerstick glucose is 354 mg/dL. A point-of-care lactate is 4.1 mmol/L. Vital signs are notable for BP 88/57, HR 105, RR 20, and T 101.7°F. In addition to broad-spectrum antibiotics and IV fluids, what is the MOST APPROPRIATE initial step in management? A Administer tetanus toxoid B Bedside ultrasound C Hyperbaric oxygen therapy D Surgical consultation

The answer is D. (Chapter 93) In Fournier's gangrene, urgent surgical consultation is of critical importance in the treatment plan. The other key elements are aggressive fluid resuscitation and broad-spectrum antibiotics. The diagnosis of Fournier's gangrene is typically made clinically, and computed tomography or other imaging should not delay operative intervention, since any delay potentially allows for worsening of the rapid progression of necrosis.

An acute coronary syndrome in which chest pain is of new onset, increasing severity, or occurs at rest, and cardiac biomarkers are not elevated.

UNSTABLE ANGINA

Who should NOT get rate controlling agents in Afib

patients with Wolff-Parkinson-White (WPW) syndrome


Kaugnay na mga set ng pag-aaral

Свідомість як фундаментальна категорія філософії та психології

View Set

OB Exam II - Postpartum and Women's Health Unit

View Set

Musculoskeletal or Articular Dysfunction

View Set

International Politics Quiz (Up to 6) Questions

View Set

Sociology Midterm #3 (Chapters 8 & 9)

View Set

PMI Project Management Processes

View Set

Chapter 53: Nursing Management: Patients With Burn Injury

View Set